Você está na página 1de 10

1o Treinamento Olı́mpico para Competições

Internacionais
Ana Paula Chaves
apchaves@ufg.br

• O Quarteto Fantástico: Euler, Fermat, Wilson e Chinês dos Restos •

1. Euler vamos mostrar uma propriedade essencial da


função φ: a sua multiplicatividade.
Dado n ∈ N, denotamos por φ(n) a quan-
tidade de números entre 1 e n que são cop-
Teorema 1.1. Sejam m, n ∈ N, tais que
rimos com n. Esta função, é conhecida por
mdc(m, n) = 1. Então, φ(mn) = φ(m)φ(n).
função totiente de Euler ou apenas função φ
de Euler.
Demonstração: Disponha os inteiros de
Exemplo 1.1. Calcule φ(6), φ(11) e φ(18) 1 até mn em m linhas e n colunas, como
Solução: Como os únicos números coprimos abaixo:
com 6, menores que ele, são 1 e 5, temos
1 2 ··· n
φ(6) = 2. Nos coprimos com 11, temos 1,
n+1 n+2 ··· 2n
2, 3, 4, 5, 6, 7, 8, 9 e 10, o que nos dá
.. .. .. ..
φ(11) = 10. Para 18, os coprimos são 1, 5, 7, . . . .
11, 13 e 17, donde φ(18) = 6. n(m − 1) + 1 n(m − 1) + 2 · · · mn

Existem φ(mn) números coprimos com mn


Exemplo 1.2. Dado p um número primo e na tabela acima, e como mdc(m, n) = 1,
n um inteiro positivo, calcule φ(pn ). temos mdc(a, mn) = 1 ⇔ mdc(a, m) = 1
Solução: Primeiro, note que entre 1 e p, ex- e mdc(a, n) = 1. Vejamos em quais colunas
istem exatamente p − 1 coprimos com p, já devemos procurar tais coprimos. Primeiro,
que não há múltiplos de p nesse intervalo, e note que se 1 ≤ i ≤ n é tal que mdc(i, n) =
n
isso nos dá φ(p) = p − 1. Para φ(p ), ob- d > 1, então todos os elementos da i−ésima
n
serve que entre 1 e p os únicos que não são coluna também não serão coprimos com n,
coprimos com p são os seus múltiplos, i.e. os pois (ki + n, i) = (n, i), donde não são cop-
n−1 n−1
números p, 2p, . . . , (p )p, nos dando p rimos com mn. Assim, os coprimos com mn
n n n−1
números. Assim, φ(p ) = p − p . estarão nas φ(n) colunas cujos primeiros el-
Como você deve imaginar, calcular a ementos são coprimos com n. Agora, tome
função φ para valores “grandes” de n, verif- uma dessas colunas, digamos a i−ésima, tal
icando manualmente quais dos seus anteces- que mdc(i, n) = 1. Os elementos desta são
sores são coprimos com o mesmo, não é nada da forma kn + i, com 0 ≤ k ≤ m − 1,
prático. No sentido de facilitar essa conta, onde temos m números que deixam restos

1
O Quarteto Fantástico: Euler, Fermat, Wilson e Chinês dos Restos Ana Paula Chaves

distintos quando divididos por m, i.e. den- que ami também é coprimo com n, donde seu
tre eles temos φ(m) coprimos com m (lem- resto na divisão por n também o é, e assim
bre que mdc(a, m) = mdc(r, m), para r o ∃ j, com 1 ≤ j ≤ φ(n), tal que ami ≡ mj
resto da divisão de a por m). Portanto, (mod n). Basta-nos então mostrar que os
em cada uma das φ(n) colunas selecionadas, ami ’s são incongruentes dois-a-dois, para que
temos φ(m) coprimos com m e n, donde a afirmação esteja demonstrada. Supondo
pelo princı́pio multiplicativo temos φ(mn) = que ami ≡ aml (mod n), temos
φ(m)φ(n). 
ami ≡ aml (mod n) ⇒ n|a(mi − ml )
Exemplo 1.3. Encontre uma fórmula para
φ(n), se a decomposição de n em fatores pri-
mos é dada por pa11 pa22 · · · pakk .
⇒ n|mi − ml ,
Solução: Pelo Teorema 1.1, temos
mas como −n + 1 < mi − ml < n − 1, o
φ(n) = φ(pa11 )φ(pa22 ) · · · φ(pakk )
único múltiplo de n neste intervalo é o zero,
k
Y
Ex. 1.2 e portanto mi = ml . Com essa afirmação
= piai −1 (pi − 1).
i=1 mostramos que, para cada i, ∃! j, tal que
Podemos também utilizar a seguinte fórmula, ami ≡ mj (mod n). Agora, multiplicando
que não precisa das potências ai ’s, mas ape- todas as congruências, obtemos
nas dos primos que dividem n:
k  
Y
ai 1 aφ(n) m1 · · · mphi(n) ≡ m1 · · · mphi(n) (mod n),
φ(n) = pi 1 −
i=1
p1
Y k 
1
 e como mdc(m1 · · · mphi(n) , n) = 1, con-
= n 1− . seguimos o desejado: aφ(n) ≡ 1 (mod n). 
i=1
p1
Este resultado nos permite simplificar
Agora estamos preparados para enunciar e
drasticamente o trabalho que temos ao cal-
demonstrar o Teorema de Euler (também
cular os resı́duos de potências grandes, como
conhecido como Teorema de Euler-Fermat).
vamos ver em alguns exemplos a seguir, e
Teorema 1.2 (Euler). Sejam a e n inteiros também “construir” alguns números com cer-
positivos, relativamente primos. Então tas propriedades.

aφ(n) ≡ 1 (mod n).


Exemplo 1.4. Mostre que existem infinitos
Demonstração: Denote por m1 , m2 , . . . , mφ(n) números da forma 20 . . . 09 que são múltiplos
o conjunto de números entre 1 e n, que de 2009.
são coprimos com n. Agora, considere
am1 , am2 , . . . , amφ(n) . Vamos mostrar que, Solução: Primeiro, note que o problema
módulo n, estes dois conjuntos são iguais a equivale a mostrar que existem infinitos k ∈
menos de ordem. Com efeito, primeiro, note N, tais que 2 · 10k + 9 é múltiplo de 2009. Isto

Pg. 2
O Quarteto Fantástico: Euler, Fermat, Wilson e Chinês dos Restos Ana Paula Chaves

é, Solução: Entre 0 e 1, temos exatamente 8


frações irredutı́veis, cujo denominador é 30:
2 · 10k + 9 ≡ 0 (mod 2009)
1 7 11 13 17 19 23 29
k , , , , , , e ,
2 · 10 + 9 ≡ 2009 (mod 2009) 30 30 30 30 30 30 30 30
2 · 10k ≡ 2000 (mod 2009) cujos numeradores são os φ(30) =
φ(2)φ(3)φ(5) = 1 · 2 · 4 = 8 números menores
(1.1) 10k ≡ 103 (mod 2009)
que 30 e coprimos com 30. A soma destas
(1.2) 10k−3 ≡ 1 (mod 2009),
frações é igual a 4. Agora, note que temos
onde todos os passos dados são reversı́veis, mais 8 frações entre 1 e 2, dadas pela lista
e as congruências (1.1) e (1.2) são válidas acima adicionando 1 à cada elemento. Por
já que (2, 2009) = (1000, 2009) = 1. As- exemplo,
sim, traduzimos o nosso problema em encon- 19 49
1+ = ∈ (1, 2).
trar infinitos k ∈ N, que satisfaçam (1.2). 30 30
Por outro lado, usando o nosso Teorema 1.2, A soma destas 8 novas frações é 4 + 8 · 1.
como (10, 2009) = 1, sabemos que Procedendo da mesma forma para todos os
demais intervalos, vamos obter uma soma to-
φ(2009)
10 ≡1 (mod 2009) tal igual à: 4 · 10 + 8 · (1 + 2 + · · · + 9) = 400.

⇒ 10t·φ(2009) ≡ 1 (mod 2009), 1.1. Problemas Propostos.

para todo t ∈ N. Assim, basta tomarmos Problema 1.1.1 (IMO 1991). Seja n > 6
k = k(t) = t · φ(2009) + 3 para termos (1.2) um inteiro e sejam a1 , a2 , . . . , ak todos os
para infinitos k(t) ∈ N. inteiros positivos, menores que n, coprimos
com n. Se
2019
Exemplo 1.5. Encontre o resto de 5 +
19 1986
quando dividido por 9. a2 − a1 = a3 − a2 = · · · = ak − ak?1 > 0,

mostre que n tem que ser um primo ou uma


Solução: Temos que φ(9) = 6, e como 5 e
potência de 2.
19 são coprimos com 9, podemos usar o Teo-
rema de Euler para conseguir, Problema 1.1.2. Encontre os três últimos
dı́gitos de 401402 .
52019 + 191986 = (56 )336 53 + (196 )331
≡ 53 + 1 (mod 9) Problema 1.1.3. Prove que para todo in-
teiro positivo n, n 6= 2 e n 6= 6, temos
⇒ 52019 + 191986 ≡ 0 (mod 9). √
φ(n) ≥ n.
Exemplo 1.6 (AIME 1992). Encontre Problema 1.1.4. Mostre que existem infini-
a soma de todos os números racionais, tos inteiros positivos n tais que
menores que 10, cujo denominador é 30 n
quando escritos de forma irredutı́vel. φ(n) = .
3

Pg. 3
O Quarteto Fantástico: Euler, Fermat, Wilson e Chinês dos Restos Ana Paula Chaves

Problema 1.1.5 (Coreia 1998). Para um in- Demonstração: Basta fazer, no Teorema
teiro positivo n, seja ψ(n) a quantidade de de Euler, m = p, e a congruência segue. 
primos que divide n. Mostre que se φ(n) di- A congruência 2.1 nos dá uma condição
vide n − 1 e ψ(n) ≤ 3, então n é primo. necessária, mas não suficiente para que um
número seja primo. Como podemos ver no
Problema 1.1.6 (OBM 1991). Demonstre
exemplo a seguir:
que existem infinitos múltiplos de 1991 que
são da forma 19999 . . . 99991. Exemplo 2.1. Prove que para todo a inteiro,
temos a561 ≡ a (mod 561).
Problema 1.1.7 (Mandelbrot 1994). Qua-
trocentas pessoas estão de pé em um cı́rculo. Solução: Primeiro, note que 561 = 3 × 11 ×
Você marca uma pessoa, então pula k pes- 17. Portanto, o problema equivale a mostrar
soas, e marca outra, pula k, e assim por di- que a(a560 − 1) é divisı́vel por 3, 11 e 17, para
ante, continuando até que você marque uma todo inteiro a. Com efeito,
pessoa pela segunda vez. Para quantos val- • Se 3 | a, não temos o que fazer. Caso 3 ∤ a,
ores positivos de k, menores que 400, todas as pelo PTF, conseguimos:
pessoas do cı́rculo serão marcadas pelo menos
a2 ≡ 1 (mod 3) ⇒ a560 = (a2 )280 ≡ 1 (mod 3)
uma vez?
∴ 3 | a560 − 1.
Problema 1.1.8 (Romênia 1997). Seja a >
• Se 11 | a, não temos o que fazer. Caso
1 um inteiro. Mostre que o conjunto
11 ∤ a, pelo PTF, conseguimos:
S = {a2 + a − 1, a3 + a2 − 1, a4 + a3 − 1, . . .}, a10 ≡ 1 (mod 11) ⇒ a560 = (a10 )56 ≡ 1 (mod 11)

contém um subconjunto infinito cujos ele- ∴ 11 | a560 − 1.


mentos são dois a dois coprimos. • Se 17 | a, não temos o que fazer. Caso
17 ∤ a, pelo PTF, conseguimos:
2. Fermat
a16 ≡ 1 (mod 17) ⇒ a560 = (a16 )35 ≡ 1 (mod 17)
O Pequeno Teorema de Fermat, como é
∴ 17 | a560 − 1.
conhecido o resultado a seguir, é um caso es-
pecial do Teorema de Euler (1.2), visto na 
seção anterior. Números compostos que satisfazem o Pe-
queno Teorema de Fermat (2.1), tais como
Teorema 2.1 (Fermat). Sejam p primo e o 561, são conhecidos como números de
a ∈ N, então, Carmichael. Na realidade, existem infinitos
números de Carmichael e 561 é o menor de-
(2.1) ap ≡ a (mod p).
les.
Mais ainda, se p ∤ a, temos
Exemplo 2.2. Mostre que não existe inteiro
p−1
a ≡ 1 (mod p). x tal que 103 | x3 − 2.

Pg. 4
O Quarteto Fantástico: Euler, Fermat, Wilson e Chinês dos Restos Ana Paula Chaves

Solução: Suponha que ∃ x ∈ Z, satis- Agora, suponha que para algum n ∈ Z ten-
3 3
fazendo 103 | x − 2. Então, temos x ≡ 2 hamos q | np − p. Então
(mod 103) ⇒ x102 ≡ 234 (mod 103) ∗ . Como 2
np ≡ p 6≡ 1 (mod q) e np ≡ pp ≡ 1 (mod q)∗ .
103 é primo, tal condição de divisibilidade
nos diz que 103 ∤ x, e Temos as condições Como q ∤ n, pelo PTF temos nq−1 ≡ 1
para aplicar o Pequeno Teorema de Fermat (mod q)∗∗ . Também temos que q 6≡ 1
102
satisfeitas, nos dando x ≡ 1 (mod 103) ∗∗
. (mod p ), o que nos dá p2 ∤ q − 1 e com isso
2

Assim, por (∗) e (∗∗), concluimos que 234 ≡ 1 que (p2 , q − 1) | p. Utilizando o Teorema de
(mod 103). Por outro lado, temos, Bezout neste último fato, existem x, y ∈ Z,
tais que
210 ≡ −6 (mod 103)
⇒ 230 ≡ −63 ≡ −216 ≡ −10 (mod 103) x · p2 + y · (q − 1) = p.
⇒ 234 ≡ −10 · 24 ≡ 46 (mod 103)
Assim,
34
∴ 2 ≡ 46 (mod 103),
2
np = (np )x (nq−1 )y ≡ 1x 1y (mod q)
uma contradição. Portanto, não existe tal x.
∴ np ≡ 1 (mod q) e np ≡ p (mod q)
Exemplo 2.3 (IMO 2003). Seja p um primo
ı́mpar. Mostre que existe um primo q tal que, ⇒ p ≡ 1 (mod q),
para todo n, o número np − p não é divisı́vel
por q. um absurdo. Portanto, q ∤ np − p, para todo
n ∈ Z.
Solução: Considere,
pp − 1
N= = pp−1 + pp−2 + · · · + p + 1.
p−1
2.1. Problemas Propostos.
Então, N ≡ p + 1 (mod p2 ) ⇒ N 6≡ 1
(mod p2 ). Desta forma, tome q como o primo Problema 2.1.1. Prove que se a e b são
que divide N, tal que q 6≡ 1 (mod p2 ). De inteiros quaisquer e p é um número primo,
fato, tal primo existe, por caso contrário então (a + b)p ≡ ap + bp (mod p).
terı́amos N ≡ 1 (mod p2 ). Como q | N,
Problema 2.1.2. Encontre todos os
então q |pp − 1 ⇒ pp ≡ 1 (mod q). Porém, p2
números primos p tais que 5 + 1 é divisı́vel
temos que p 6≡ 1 (mod q), pois caso contráro
por p.
terı́amos,
Problema 2.1.3. . (1) Seja a um inteiro
N ≡ 1+1+···+1 = p (mod q)
positivo. Mostre que qualquer fator primo
maior que 2 de a2 + 1 é da forma 4m + 1.
⇒ N ≡ 1 (mod q),
(2) Prove que existem infinitos primos da
um absurdo já que q | N. forma 4m + 1.

Pg. 5
O Quarteto Fantástico: Euler, Fermat, Wilson e Chinês dos Restos Ana Paula Chaves

Problema 2.1.4. Mostre que, para todo 3. Wilson


primo p > 5, o número
Apesar de levar o nome do matemático J.
111
| {z. . . 1}, Wilson, o resultado a seguir já era conhecido
p−1 por G. W. Leibniz um século antes. Na ver-
é divisı́vel por p. dade, Wilson apenas propôs o resultado, que
foi provado por J.-L. Lagrange em 1773.
Problema 2.1.5 (OCM 2007). Ache todos
Diferente do PTF, o Teorema de Wilson
os primos p, q tais que pp + q q + 1 é múltiplo
nos dá um teste de primalidade, ou seja, uma
de pq.
condição necessária e suficiente para que um
Problema 2.1.6. Mostre que, se p é um determinado número seja primo.
primo congruente à 3 módulo 4, então não
existe a tal que Teorema 3.1 (Wilson). p é primo se, e so-
mente se, (p − 1)! ≡ −1 (mod p).
a2 ≡ −1 (mod p).
Ideia da demonstração: Primeiro, observe
Problema 2.1.7. Mostre que dado qualquer
que, se p é composto, então todos os divi-
p primo, pp+1 + (p + 1)p não é um quadrado
sores deste são menores que p−1, donde cada
perfeito.
um deles divide (p − 1)!, sendo impossı́vel
Problema 2.1.8 (Ucrânia 1997). Encontre que tenhamos (p − 1)! ≡ −1 (mod p), pois
o menor inteiro n tal que, entre quaisquer isto implicaria que (p − 1)! e p são coprimos.
n inteiros, permitindo repetição, existem 18 Agora, caso p seja primo, podemos separar o
inteiros cuja soma é divisı́vel por 18. conjunto dos restos 2, 3, . . . , p − 2 em pares
de números cujo produto é ≡ 1 (mod p).
Problema 2.1.9 (Bulgária 1995). Encontre
Portanto, o produto (p − 1)! ≡ (p − 1)
a quantidade de inteiros n > 1 para os quais
(mod p). 
a25 −a é divisı́vel por n, para qualquer inteiro
a. Exemplo 3.1 (Estônia 2000). Prove que não
é possı́vel dividir qualquer conjunto de 18 in-
Problema 2.1.10 (IMO 1963). .
teiros consecutivos em dois conjuntos disjun-
(a) Encontre todos os n inteiros positivos
tos A e B tais que o produto dos elementos
tais que 7 divide 2n − 1.
de A seja igual ao produto dos elementos de
(b) Mostre que para todo n inteiro positivo,
B.
o número 2n + 1 não é divisı́vel por 7.

Problema 2.1.11 (IMO 2005). Considere a Solução: Suponha, por absurdo, que ex-
sequência a1 , a2 , . . . , dada por istam tais conjuntos, e que os 18 consecu-
tivos são n, n + 1, . . . , n + 17. Agora, note
an = 2n + 3n + 6n − 1, (n = 1, 2, ...).
que, como o produto dos elementos de A
Determine todos os inteiros positivos que são é igual ao produto dos elementos de B, se
coprimos com todos os termos da sequência. um dos conjuntos contém um múltiplo de

Pg. 6
O Quarteto Fantástico: Euler, Fermat, Wilson e Chinês dos Restos Ana Paula Chaves

19, o outro também deve conter. Porém, en- Portanto,


tre 18 inteiros consecutivos, não existem dois
(n − 1)! · (−1)p−n · (p − n)! ≡ −1 (mod p),
múltiplos de 19. Deste modo, nenhum dos
dois conjuntos possui elementos divisı́veis por e multiplicando em ambos os lados por
19. Denote por r o resto da divisão do pro- (−1)p−n , observando que p é ı́mpar, final-
duto de todos os elementos de A, PA , por mente conseguimos
19, que é o mesmo do produto dos elemen- (n−1)!·(p−n)! ≡ (−1)n−p+1 ≡ (−1)n (mod p).
tos de B, PB , já que supomos que são iguais.
Exemplo 3.3 (Áustria-Polônia 1996).
Assim,
Mostre que não existem inteiros não nega-
r · r = PA · PB tivos k e m tais que k! + 48 = 48 · (k + 1)m .
≡ n(n + 1) · · · (n + 17) (mod 19)
Solução: Suponha, por absurdo, que exis-
≡ 1 · 2 · 3 · · · 18 (mod 19) tam tais k e m. Então, devemos ter que
≡ (19 − 1)! (mod 19) 48 | k!, donde k deve ser pelo menos 6. Ob-
≡ −1 (mod 19) servando que (6! + 48)/48 = 46 6= 7m , então
k ≥ 7. Da mesma forma, conseguimos que
∴ r 2 ≡ −1 (mod 19),
k 6= 7, nos dando k ≥ 8. Note que, nesse
a
onde a 3 linha vem do fato do conjunto de 18 caso, 6 | k!/48, ou seja 1 = (6, k!/48 + 1) =
elementos consecutivos não possuir múltiplos 1 = (6, (k + 1)m ), e assim 6 e k + 1 são cop-
de 19, e 5a é consequência do Teorema de rimos.
2 18
Wilson. Assim, r ≡ −1 (mod 19) ⇒ r ≡ Se tivermos k + 1 composto, então existe
(−1)9 ≡ −1 (mod 19), mas isso contraria o um primo p > 3, já que ambos 2 e 3 não
Pequeno Teorema de Fermat e conseguimos dividem k + 1, tal que p | k + 1. Por outro
um absurdo. Portanto, tais conjuntos não lado, como 48 = 24 · 3, temos que p | k! mas
existem. p ∤ k! + 48, um absurdo já que p | k + 1.
Portanto, k + 1 é primo e, pelo Teorema de
Exemplo 3.2. Mostre que, se p é um primo
Wilson, temos que k +1 | k!+1. Combinando
ı́mpar, então para todo inteiro positivo n < p,
este último fato com k + 1 | k! + 48, dado no
temos
enunciado, conseguimos k+1 | 47 ⇒ k = 46.
(n − 1)!(p − n)! ≡ (−1)n (mod p). Agora, resta-nos mostrar que 46!/48 + 1
não é uma potência de 47. Complete essa
Solução: Pelo Teorema de Wilson, sabemos
parte! :)
que (p − 1)! ≡ −1 (mod p), e como n < p,
reescrevemos esta congruência como, 3.1. Problemas Propostos.

(n−1)! · n· (n+ 1) · · · (p −1) ≡ −1 (mod p), Problema 3.1.1. Mostre que para todo p
primo e todo 0 ≤ k ≤ p − 1, temos
que equivale à,  
p−1
≡ (−1)k (mod p).
(n − 1)! · (−1)(p − n) · · · (−1) ≡ −1 (mod p). k

Pg. 7
O Quarteto Fantástico: Euler, Fermat, Wilson e Chinês dos Restos Ana Paula Chaves

Problema 3.1.2. Encontre todos os inteiros europeus, muçulmanos e etc), contribuiram


n
positivos a e n tais que a = (a − 1)! + 1. para o desenvolvimento do resultado, dado a
seguir.
Problema 3.1.3 (Irlanda 1996). Para cada
n inteiro positivo, encontre o máximo divisor Teorema 4.1 (Chinês dos Restos). Sejam
comum entre n! + 1 e (n + 1)!. b1 , b2 , . . . , bk inteiros e m1 , m1 , . . . , mk copri-
mos dois-a-dois. Então, o sistema de con-
Problema 3.1.4 (Seletiva da IMO -
gruências
Romênia 1986). Seja p ≥ 3 um primo e seja
σ uma permutação de {1, 2, . . . , p−1}. Prove x ≡ b1 (mod m1 )
que existem i 6= j tais que p | iσ(i) − jσ(j). x ≡ b2 (mod m2 )
..
Problema 3.1.5. Seja p um primo ı́mpar. .
Mostre que
x ≡ bk (mod mk ),
p+1
2 2 2
1 · 3 · · · (p − 2) ≡ (−1) 2 (mod p)
admite solução, que é única módulo M =
e m1 m2 · · · mk . A saber,
p+1
22 · 42 · · · (p − 1)2 ≡ (−1) 2 (mod p). x0 = M1 X1 b1 + M2 X2 b2 + · · · + Mk Xk bk ,
Problema 3.1.6. Prove que se a e b são onde Mi := M/mi e Xi Mi ≡ 1 (mod mi ), é
inteiros quaisquer e p é um número primo, solução.
então (a + b)p ≡ ap + bp (mod p).
Para os curiosos, a demonstração deste
Problema 3.1.7. Mostre que a sequência fato pode ser encontrada na excelente re-
2
an = (n!) − n! + 1 contém infinitos números ferência [2].
compostos.
Exemplo 4.1 (AIME 2012). Para um
Problema 3.1.8. Prove que, se p é um
inteiro positivo p, dizemos que n ∈
primo ı́mpar, então o resto da divisão de
N é p−seguro, se n difere, em valor
(p − 1)! por p(p − 1) é p − 1.
absoluto, por mais de 2 de todos os
4. O Teorema Chinês dos Restos múltiplos de p. Por exemplo. o
conjunto dos números 10−seguros é
A necessidade dos chineses, pelo menos há
{3, 4, 5, 6, 7, 13, 14, 15, 16, 17, 23, . . .}. En-
200 anos A.C, de elaborar um calendário sat-
contre quantos são os inteiros positivos
isfazendo uma série de condições relacionadas
menores que ou iguais à 10000 que são
à astronomia, deu origem ao famoso Teorema
simultâneamente 7−seguros, 11−seguros e
Chinês dos Restos. Congruências lineares
13−seguros.
simultâneas, foram primeiro estudadas por
Sun Zi, matemático chinês responsável pela Solução: Observe que n é p−seguro se, e so-
primeira abordagem do problema. Depois mente se, o seu resı́duo módulo p é maior que
disso, vários matemáticos (chineses, hindus, 2 e menor que p−2, nos dando p−5 possı́veis

Pg. 8
O Quarteto Fantástico: Euler, Fermat, Wilson e Chinês dos Restos Ana Paula Chaves

resı́duos para n módulo p. Portanto, se n sat- Primeiro, como podemos produzir k ∈ N


isfaz as condições do problema, então n pode tal que P (k) ≡ 0 (mod pi ), para algum con-
ter 2 resı́duos diferentes módulo 7, 6 resı́duos junto de n primos p1 , . . . , pn ? Primeiro, note
diferentes módulo 11 e 8 módulo 13. Pelo que se k ≡ ki (mod pi ), então P (k) ≡ P (ki )
Teorema Chinês dos Restos, sabemos que, (mod pi ), portanto, construindo soluções
qualquer que seja o trio destes resı́duos escol- particulares P (ki ) ≡ 0 (mod pi ), para todo
hido, conseguimos uma única solução módulo i, o Teorema Chinês dos Restos nos garante
7 · 11 · 13 = 1001, já que 7, 11 e 13 são dois a a existência de k ≡ ki (mod pi ), também
dois coprimos. Por exemplo, n pode ser tal para todo i, o que, pela nossa observação
que prévia, acarreta em P (k) ≡ P (ki ) ≡ 0
(mod pi ), como gostarı́amos. Passemos a
n ≡ 3 (mod 7)
construção de tais ki′ s e p′i s. Tome p0 um
n ≡ 7 (mod 11)
primo. Considere P (p0 ) = p20 + p0 + 1. Como
n ≡ 3 (mod 13), (P (p0 ), p0 ) = 1, existe um primo p1 6= p0 tal
ou seja, n ≡ 458 (mod 1001). Note que, que p1 | P (p0 ), donde P (p0 ) ≡ 0 (mod p1 ).
para este trio de resı́duos, temos 10 val- Agora, tome P (p0 p1 ) = (p0 p1 )2 + p0 p1 + 1.
ores 0 ≤ n < 10010 possı́veis, que são Novamente, (P (p0 p1 ), p0 p1 ) = 1 e existe um
{458, 1459, 2460, 3461, . . . , 9467}. Isso é in- primo p2 6∈ {p0 , p1 }, tal que p2 | P (p0 p1 ),
variante para qualquer trio escolhido, nos donde P (p0p1 ) ≡ 0 (mod p2 ). Procedendo
dando um total de 2 · 6 · 8 · 10 = 960 val- recursivamente, no n−ésimo passo, tere-
ores possı́veis para n neste intervalo. Porém, mos P (p0 p1 · · · pn−1 ) ≡ 0 (mod pn ), e assim
devemos retirar os valores entre 10001 e construı́do as n soluções particulares para
10010 que safistazem tais condições, sendo P (ki ) ≡ 0 (mod pi ). Isto conclui o prob-
eles 10006 e 10007, nos dando finalmente 958 lema.
números.
4.1. Problemas Propostos.
Exemplo 4.2 (USAMO 2008/1). Mostre
que, para cada n inteiro positivo, existem Problema 4.1.1. Seja N = 1234567 . . . 4344
k0 , k1 , . . . , kn inteiros positivos maiores que o número de 79 dı́gitos, obtido escrevendo os
1, dois-a-dois coprimos, tais que k0 k1 · · · kn − inteiros de 1 até 44 concatenados. Qual é o
1 é o produto de dois inteiros consecutivos. resto que N deixa quando dividido por 45?

Solução: Observe que o problema equivale Problema 4.1.2 (IMO 1989). Mostre que,
á mostrar que, dado n inteiro positivo, existe para todo n inteiro positivo, existem n in-
k ∈ N tal que k(k + 1) + 1 possui pelo menos teiros positivos consecutivos, nenhum dos
n + 1 divisores primos. Ou seja, basta-nos quais é potência de primo.
mostrar que o polinômio P (k) = k 2 + k + 1
produz inteiros positivos divisı́veis por quan- Problema 4.1.3. Mostre que, dados k e
tidades arbitrariamente grandes de primos. n números naturais, é possı́vel encontrar k

Pg. 9
O Quarteto Fantástico: Euler, Fermat, Wilson e Chinês dos Restos Ana Paula Chaves

números consecutivos, cada um dos quais tem 1, 2, . . . , k−1. Prove que n não divide ak (a1 −
pelo menos n divisores primos distintos. 1).

Problema 4.1.6. Um inteiro positivo n é


Problema 4.1.4 (Olimpı́ada de Mayo
chamado de auto-replicante se os últimos
2013). É possı́vel escrever 100 números
dı́gitos de n2 formam o número n. Por exem-
ı́mpares numa fila de tal forma que a soma de
plo, 25 é auto-replicante pois 252 = 625.
cada 5 adjacentes seja um quadrado perfeito
Determine todos os números auto-replicantes
e que a soma de cada 9 números adjacentes
com exatamente 4 dı́gitos.
também seja um quadrado perfeito?
Problema 4.1.7 (OBM 2017). Demonstre
Problema 4.1.5 (IMO 2009). Seja n um in- que, para todo n inteiro positivo, existem in-
teiro positivo e sejam a1 , a2 , . . . , ak , com k ≥ teiros positivos a e b, sem fatores primos em
2, inteiros distintos do conjunto {1, 2, . . . , n} comum, de modo que a2 + 2017b2 possui mais
tais que n divide ai (ai+1 − 1) para i = de n fatores primos distintos.

References
[1] Andreescu, T., Andrica, D. and Feng Z., 104 Number Theory Problems, Birkhauser, Boston, MA, 2007.
[2] F. E. Brochero Martinez, C. G. Moreira, N. C. Saldanha, E. Tengan Teoria dos Números: um passeio com
primos e outros números familiares pelo mundo inteiro, Projeto Euclides, IMPA, 2010.
[3] Carneiro, E., Campos, O. e Paiva, F. Olimpı́adas Cearenses de Matemáatica 1981-2005 (Nı́veis Júnior e
Senior), Ed. Realce, 2005.
[4] Feitosa, S. B. , Holanda, B., Lima, Y. e Magalhães, C. T. Treinamento Cone Sul 2008. Fortaleza, Ed. Re-
alce, 2010.
[5] Fomin, D. and Kirichenko, A. Leningrad Mathematical Olympiads 1987-1991, MathPro Press, Westford,
MA, 1994.
[6] Fomin, D., Genkin, S. and Itenberg, I. Mathematical Circles, Mathematical Words, Vol. 7, American
Mathematical Society, Boston, MA, 1966.
[7] Niven, I., Zuckerman, H. S. and Montgomery, H. L. An Introduction to the Theory of Numbers.
[8] Art of Problem Solving - https://artofproblemsolving.com

“[When asked why are numbers beautiful?]


It’s like asking why is Ludwig van Beethoven’s Ninth Symphony beautiful. If you don’t see why, someone can’t tell you. I know
numbers are beautiful. If they aren’t beautiful, nothing is.” - P. Erdös

Pg. 10

Você também pode gostar